Last visit was: 26 Apr 2024, 19:33 It is currently 26 Apr 2024, 19:33

Close
GMAT Club Daily Prep
Thank you for using the timer - this advanced tool can estimate your performance and suggest more practice questions. We have subscribed you to Daily Prep Questions via email.

Customized
for You

we will pick new questions that match your level based on your Timer History

Track
Your Progress

every week, we’ll send you an estimated GMAT score based on your performance

Practice
Pays

we will pick new questions that match your level based on your Timer History
Not interested in getting valuable practice questions and articles delivered to your email? No problem, unsubscribe here.
Close
Request Expert Reply
Confirm Cancel
SORT BY:
Date
Tags:
   
Show Tags
Hide Tags
Director
Director
Joined: 21 Jun 2017
Posts: 638
Own Kudos [?]: 531 [0]
Given Kudos: 4092
Location: India
Concentration: Finance, Economics
GMAT 1: 660 Q49 V31
GMAT 2: 620 Q47 V30
GMAT 3: 650 Q48 V31
GPA: 3.1
WE:Corporate Finance (Non-Profit and Government)
Send PM
Math Revolution GMAT Instructor
Joined: 16 Aug 2015
Posts: 10161
Own Kudos [?]: 16600 [0]
Given Kudos: 4
GMAT 1: 760 Q51 V42
GPA: 3.82
Send PM
Math Revolution GMAT Instructor
Joined: 16 Aug 2015
Posts: 10161
Own Kudos [?]: 16600 [0]
Given Kudos: 4
GMAT 1: 760 Q51 V42
GPA: 3.82
Send PM
Math Revolution GMAT Instructor
Joined: 16 Aug 2015
Posts: 10161
Own Kudos [?]: 16600 [0]
Given Kudos: 4
GMAT 1: 760 Q51 V42
GPA: 3.82
Send PM
Re: Math Revolution DS Expert - Ask Me Anything about GMAT DS [#permalink]
Expert Reply
[Math Revolution GMAT math practice question]

(number property) \(f(x)\) is the greatest prime factor of \(x\). If \(n\) is a positive integer less than \(10\), what is the value of \(n\)?

\(1) f(n) = f(1000)\)
\(2) f(n) = 5\)
Math Revolution GMAT Instructor
Joined: 16 Aug 2015
Posts: 10161
Own Kudos [?]: 16600 [0]
Given Kudos: 4
GMAT 1: 760 Q51 V42
GPA: 3.82
Send PM
Re: Math Revolution DS Expert - Ask Me Anything about GMAT DS [#permalink]
Expert Reply
ShankSouljaBoi wrote:
MathRevolution wrote:
MathRevolution wrote:
[Math Revolution GMAT math practice question]

(number property) If \(n\) is an integer greater than \(1\), what is the value of \(n\)?

1) \(n\) is a prime number
2) \(\frac{(n+2)}{n}\) is an integer


=>

Forget conventional ways of solving math questions. For DS problems, the VA (Variable Approach) method is the quickest and easiest way to find the answer without actually solving the problem. Remember that equal numbers of variables and independent equations ensure a solution.

Since we have 1 variable (n) and 0 equations, D is most likely to be the answer. So, we should consider each of the conditions on their own first.

Condition 1)
Since there are many prime numbers, condition 1) is not sufficient.

Condition 2)
If \(n = 1\), then \(\frac{(n+2)}{n} = 3\) is an integer.
If \(n = 2,\) then \(\frac{(n+2)}{n} = 2\) is an integer.
Since we don’t have a unique solution, condition 2) is not sufficient.

Conditions 1) & 2)
If \(n = 2\), then \(\frac{(n+2)}{n} = 2\) is an integer.
If \(n = 3\), then \(\frac{(n+2)}{n} = \frac{5}{2}\) is not integer.
If \(n\) is a prime number bigger than \(2\), \(\frac{(n+2)}{n}\) is not an integer.
Thus \(n = 2\) is the unique solution and both conditions together are sufficient.

Therefore, C is the answer.
Answer: C

If the original condition includes “1 variable”, or “2 variables and 1 equation”, or “3 variables and 2 equations” etc., one more equation is required to answer the question. If each of conditions 1) and 2) provide an additional equation, there is a 59% chance that D is the answer, a 38% chance that A or B is the answer, and a 3% chance that the answer is C or E. Thus, answer D (conditions 1) and 2), when applied separately, are sufficient to answer the question) is most likely, but there may be cases where the answer is A,B,C or E.

Please check the OA, Highlighted text is wrong. Stem alreay mention n as > 1.

Regards


You are right.
The question should be changed to the followings.

If \(n\) is a positive integer, what is the value of \(n\)?

Thank you for your comments.
Math Revolution GMAT Instructor
Joined: 16 Aug 2015
Posts: 10161
Own Kudos [?]: 16600 [1]
Given Kudos: 4
GMAT 1: 760 Q51 V42
GPA: 3.82
Send PM
Re: Math Revolution DS Expert - Ask Me Anything about GMAT DS [#permalink]
1
Kudos
Expert Reply
[Math Revolution GMAT math practice question]

(number properties) If \(n\) is a positive integer, is \(\sqrt{n+1}\) an even integer?

1) \(n\) is the product of \(2\) consecutive odd numbers
2) \(n\) is an odd number
Math Revolution GMAT Instructor
Joined: 16 Aug 2015
Posts: 10161
Own Kudos [?]: 16600 [0]
Given Kudos: 4
GMAT 1: 760 Q51 V42
GPA: 3.82
Send PM
Re: Math Revolution DS Expert - Ask Me Anything about GMAT DS [#permalink]
Expert Reply
MathRevolution wrote:
[Math Revolution GMAT math practice question]

(number property) \(f(x)\) is the greatest prime factor of \(x\). If \(n\) is a positive integer less than \(10\), what is the value of \(n\)?

\(1) f(n) = f(1000)\)
\(2) f(n) = 5\)


=>

Forget conventional ways of solving math questions. For DS problems, the VA (Variable Approach) method is the quickest and easiest way to find the answer without actually solving the problem. Remember that equal numbers of variables and independent equations ensure a solution.

Since we have \(1\) variable (\(n\)) and \(0\) equations, D is most likely to be the answer. So, we should consider each condition on its own first.

Condition 1)
Since \(1000 = 2^3*5^3, f(1000) = 5.\)
So, \(f(n) = 5\) and \(n = 5.\)
Thus, condition 1) is sufficient, since it gives a unique solution.

Condition 2)
Condition 2) is the same as condition 1).
Thus, condition 2) is also sufficient.

FYI, Tip 1) of the VA method states that D is most likely to be the answer if conditions 1) and 2) provide the same information.

Therefore, D is the answer.
Answer: D

If the original condition includes “1 variable”, or “2 variables and 1 equation”, or “3 variables and 2 equations” etc., one more equation is required to answer the question. If each of conditions 1) and 2) provide an additional equation, there is a 59% chance that D is the answer, a 38% chance that A or B is the answer, and a 3% chance that the answer is C or E. Thus, answer D (conditions 1) and 2), when applied separately, are sufficient to answer the question) is most likely, but there may be cases where the answer is A,B,C or E.
Math Revolution GMAT Instructor
Joined: 16 Aug 2015
Posts: 10161
Own Kudos [?]: 16600 [0]
Given Kudos: 4
GMAT 1: 760 Q51 V42
GPA: 3.82
Send PM
Re: Math Revolution DS Expert - Ask Me Anything about GMAT DS [#permalink]
Expert Reply
[Math Revolution GMAT math practice question]

(number properties) If \(m\) and \(n\) are positive integers, is \(m + n\) an odd number?

1) \(\frac{m}{n}\) is an even number
2) \(m\) or \(n\) is an even number
GMAT Club Legend
GMAT Club Legend
Joined: 18 Aug 2017
Status:You learn more from failure than from success.
Posts: 8020
Own Kudos [?]: 4098 [1]
Given Kudos: 242
Location: India
Concentration: Sustainability, Marketing
GMAT Focus 1:
545 Q79 V79 DI73
GPA: 4
WE:Marketing (Energy and Utilities)
Send PM
Re: Math Revolution DS Expert - Ask Me Anything about GMAT DS [#permalink]
1
Kudos
MathRevolution wrote:
[Math Revolution GMAT math practice question]

(number properties) If \(m\) and \(n\) are positive integers, is \(m + n\) an odd number?

1) \(\frac{m}{n}\) is an even number
2) \(m\) or \(n\) is an even number



#1:
m/n = even no
m=6, n= 3 m/n= 2 ; m+n= 6+3= 9 sufficient

m=4, n=2 ; m/n= 2 ; m+n=4+2 = 6 not sufficient

#1 not sufficient

#2 :
m or n is an even no

property of odd no in addition : even+ odd = odd no
so since m & n are + integers so either of m or n being even other has to be odd integer hence we would get m+n= odd only

sufficient

IMO B
Tutor
Joined: 12 Oct 2010
Status:GMATH founder
Posts: 893
Own Kudos [?]: 1356 [0]
Given Kudos: 56
Send PM
Re: Math Revolution DS Expert - Ask Me Anything about GMAT DS [#permalink]
Expert Reply
MathRevolution wrote:
[Math Revolution GMAT math practice question]

(number properties) If \(m\) and \(n\) are positive integers, is \(m + n\) an odd number?

1) \(\frac{m}{n}\) is an even number
2) \(m\) or \(n\) is an even number

\(m,n\,\,\, \geqslant 1\,\,\,{\text{ints}}\,\,\,\,\left( * \right)\)

\(m + n\,\,\,\,\mathop = \limits^? \,\,{\text{odd}}\,\,\,\,\,\mathop \Leftrightarrow \limits^{\left( * \right)} \,\,\,\,\boxed{\,\,\,?\,\,\,:\,\,\,\left( {m\,\,{\text{odd}}\,,\,\,n\,\,{\text{even}}} \right)\,\,\,{\text{or}}\,\,\,{\text{vice - versa}\,\,}\,\,}\)


\(\left( 1 \right)\,\,\,\frac{m}{n} = {\text{even}}\,\,\,\,\left\{ \begin{gathered}\\
\,{\text{Take}}\,\,\left( {m,n} \right) = \left( {2,1} \right)\,\,\,\, \Rightarrow \,\,\,\left\langle {{\text{YES}}} \right\rangle \,\, \hfill \\\\
\,{\text{Take}}\,\,\left( {m,n} \right) = \left( {4,2} \right)\,\,\,\, \Rightarrow \,\,\,\left\langle {{\text{NO}}} \right\rangle \,\, \hfill \\ \\
\end{gathered} \right.\)

\(\left( 2 \right)\,\,\,m\,\,{\text{even}}\,\,\,{\text{or}}\,\,\,n\,\,{\text{even}}\,\,\,\,\left\{ \begin{gathered}\\
\,\left( {\operatorname{Re} } \right){\text{Take}}\,\,\left( {m,n} \right) = \left( {2,1} \right)\,\,\,\, \Rightarrow \,\,\,\left\langle {{\text{YES}}} \right\rangle \,\, \hfill \\\\
\,\left( {\operatorname{Re} } \right){\text{Take}}\,\,\left( {m,n} \right) = \left( {4,2} \right)\,\,\,\, \Rightarrow \,\,\,\left\langle {{\text{NO}}} \right\rangle \,\, \hfill \\ \\
\end{gathered} \right.\,\,\,\,\,\,\,\,\,\,\,\,\,\, \Rightarrow \,\,\,\,\,\,\,\,\,\,\,\,\,\left( {\text{E}} \right)\)


This solution follows the notations and rationale taught in the GMATH method.

Regards,
Fabio.

P.S.: "A or B" means "only A", "only B" or BOTH.
Tutor
Joined: 12 Oct 2010
Status:GMATH founder
Posts: 893
Own Kudos [?]: 1356 [0]
Given Kudos: 56
Send PM
Re: Math Revolution DS Expert - Ask Me Anything about GMAT DS [#permalink]
Expert Reply
MathRevolution wrote:
[Math Revolution GMAT math practice question]

(number properties) If \(n\) is a positive integer, is \(\sqrt{n+1}\) an even integer?

1) \(n\) is the product of \(2\) consecutive odd numbers
2) \(n\) is an odd number

Beautiful problem, Max. Congrats (and kudos)!

\(n \geqslant 1\,\,\,\operatorname{int}\)

\(\sqrt {n + 1} \,\,\,\mathop = \limits^? \,\,{\text{even}}\,\,\,\,\,\,\,\mathop \Leftrightarrow \limits^{\left( * \right)} \,\,\,\,\,\,\boxed{\,\,n + 1\,\,\,\mathop = \limits^? \,\,\,{{\left( {{\text{even}}} \right)}^2}\,\,}\)


\(\left( 1 \right)\,\,\,n = \left( {2M - 1} \right)\left( {2M + 1} \right) = {\left( {2M} \right)^2} - {\left( 1 \right)^2}\,\,\,\,\,\left[ {M\,\,\operatorname{int} \,} \right]\,\,\,\,\,\,\,\,\, \Rightarrow \,\,\,\,\,\,\,\,\,n + 1 = {\left( {2M} \right)^2}\,\,\,,\,\,\,\,M\,\,\operatorname{int} \,\,\,\,\,\,\,\,\,\, \Rightarrow \,\,\,\,\,\,\,\,\,\left\langle {{\text{YES}}} \right\rangle \,\)

\(\left( 2 \right)\,\,\,\left\{ \matrix{\\
\,{\rm{Take}}\,\,n = 1\,\,\,\,\, \Rightarrow \,\,\,\,\,\left\langle {{\rm{NO}}} \right\rangle \,\, \hfill \cr \\
\,{\rm{Take}}\,\,n = 3\,\,\,\,\, \Rightarrow \,\,\,\,\,\left\langle {{\rm{YES}}} \right\rangle \,\, \hfill \cr} \right.\,\,\)


This solution follows the notations and rationale taught in the GMATH method.

Regards,
Fabio.
GMAT Club Legend
GMAT Club Legend
Joined: 18 Aug 2017
Status:You learn more from failure than from success.
Posts: 8020
Own Kudos [?]: 4098 [0]
Given Kudos: 242
Location: India
Concentration: Sustainability, Marketing
GMAT Focus 1:
545 Q79 V79 DI73
GPA: 4
WE:Marketing (Energy and Utilities)
Send PM
Re: Math Revolution DS Expert - Ask Me Anything about GMAT DS [#permalink]
fskilnik wrote:
MathRevolution wrote:
[Math Revolution GMAT math practice question]

(number properties) If \(m\) and \(n\) are positive integers, is \(m + n\) an odd number?

1) \(\frac{m}{n}\) is an even number
2) \(m\) or \(n\) is an even number

\(m,n\,\,\, \geqslant 1\,\,\,{\text{ints}}\,\,\,\,\left( * \right)\)

\(m + n\,\,\,\,\mathop = \limits^? \,\,{\text{odd}}\,\,\,\,\,\mathop \Leftrightarrow \limits^{\left( * \right)} \,\,\,\,\boxed{\,\,\,?\,\,\,:\,\,\,\left( {m\,\,{\text{odd}}\,,\,\,n\,\,{\text{even}}} \right)\,\,\,{\text{or}}\,\,\,{\text{vice - versa}\,\,}\,\,}\)


\(\left( 1 \right)\,\,\,\frac{m}{n} = {\text{even}}\,\,\,\,\left\{ \begin{gathered}\\
\,{\text{Take}}\,\,\left( {m,n} \right) = \left( {2,1} \right)\,\,\,\, \Rightarrow \,\,\,\left\langle {{\text{YES}}} \right\rangle \,\, \hfill \\\\
\,{\text{Take}}\,\,\left( {m,n} \right) = \left( {4,2} \right)\,\,\,\, \Rightarrow \,\,\,\left\langle {{\text{NO}}} \right\rangle \,\, \hfill \\ \\
\end{gathered} \right.\)

\(\left( 2 \right)\,\,\,m\,\,{\text{even}}\,\,\,{\text{or}}\,\,\,n\,\,{\text{even}}\,\,\,\,\left\{ \begin{gathered}\\
\,\left( {\operatorname{Re} } \right){\text{Take}}\,\,\left( {m,n} \right) = \left( {2,1} \right)\,\,\,\, \Rightarrow \,\,\,\left\langle {{\text{YES}}} \right\rangle \,\, \hfill \\\\
\,\left( {\operatorname{Re} } \right){\text{Take}}\,\,\left( {m,n} \right) = \left( {4,2} \right)\,\,\,\, \Rightarrow \,\,\,\left\langle {{\text{NO}}} \right\rangle \,\, \hfill \\ \\
\end{gathered} \right.\,\,\,\,\,\,\,\,\,\,\,\,\,\, \Rightarrow \,\,\,\,\,\,\,\,\,\,\,\,\,\left( {\text{E}} \right)\)


This solution follows the notations and rationale taught in the GMATH method.

Regards,
Fabio.

P.S.: "A or B" means "only A", "only B" or BOTH.



fskilnik

#2 says m or n even ; why have you taken both as even while proving statement as insufficient?
Tutor
Joined: 12 Oct 2010
Status:GMATH founder
Posts: 893
Own Kudos [?]: 1356 [0]
Given Kudos: 56
Send PM
Re: Math Revolution DS Expert - Ask Me Anything about GMAT DS [#permalink]
Expert Reply
Archit3110 wrote:
#2 says m or n even ; why have you taken both as even while proving statement as insufficient?

Hi Archit3110 ,

Thank you for your interest in my solution.

As I explained in my post scriptum (PS), the word "OR" has not the everyday common use of "exclusive or".

In other words, when it is given that "m is even or n is even", there are three possibilities available:

(i) m is even and n is not even
(ii) m is not even and n is even
(iii) m is even and n is also even

Regards and success in your studies,
Fabio.
Math Revolution GMAT Instructor
Joined: 16 Aug 2015
Posts: 10161
Own Kudos [?]: 16600 [0]
Given Kudos: 4
GMAT 1: 760 Q51 V42
GPA: 3.82
Send PM
Re: Math Revolution DS Expert - Ask Me Anything about GMAT DS [#permalink]
Expert Reply
MathRevolution wrote:
[Math Revolution GMAT math practice question]

(number properties) If \(n\) is a positive integer, is \(\sqrt{n+1}\) an even integer?

1) \(n\) is the product of \(2\) consecutive odd numbers
2) \(n\) is an odd number


=>

Forget conventional ways of solving math questions. For DS problems, the VA (Variable Approach) method is the quickest and easiest way to find the answer without actually solving the problem. Remember that equal numbers of variables and independent equations ensure a solution.

The first step of the VA (Variable Approach) method is to modify the original condition and the question. We then recheck the question.

The question is equivalent to asking if \(\sqrt{n+1} = 2k\) for some positive integer \(k\).
\(\sqrt{n+1} = 2k\)
\(=> n+1 = 4k^2\)
\(=> n = 4k^2-1\)
\(=> n = (2k-1)(2k+1)\)
\(n\) is a product of two consecutive odd integers.
Thus, condition 1) is sufficient.

Condition 2)
If \(n = 3\), then \(\sqrt{3+1} = \sqrt{4}=2\) and the answer is ‘yes’.
If \(n = 1\), then \(\sqrt{1+1} = 2\) is not an integer and the answer is ‘no’.
Condition 2) is not sufficient.

Therefore, A is the answer.
Answer: A
Math Revolution GMAT Instructor
Joined: 16 Aug 2015
Posts: 10161
Own Kudos [?]: 16600 [0]
Given Kudos: 4
GMAT 1: 760 Q51 V42
GPA: 3.82
Send PM
Re: Math Revolution DS Expert - Ask Me Anything about GMAT DS [#permalink]
Expert Reply
[Math Revolution GMAT math practice question]

(number properties) If \(m\) and \(n\) are positive integers, is \(3^{4m+2}+n\) divisible by \(5\)?

\(1) m=3\)
\(2) n=1\)
Math Revolution GMAT Instructor
Joined: 16 Aug 2015
Posts: 10161
Own Kudos [?]: 16600 [0]
Given Kudos: 4
GMAT 1: 760 Q51 V42
GPA: 3.82
Send PM
Math Revolution DS Expert - Ask Me Anything about GMAT DS [#permalink]
Expert Reply
MathRevolution wrote:
[Math Revolution GMAT math practice question]

(number properties) If \(m\) and \(n\) are positive integers, is \(m + n\) an odd number?

1) \(\frac{m}{n}\) is an even number
2) \(m\) or \(n\) is an even number


=>

Forget conventional ways of solving math questions. For DS problems, the VA (Variable Approach) method is the quickest and easiest way to find the answer without actually solving the problem. Remember that equal numbers of variables and independent equations ensure a solution.

Since we have \(2\) variables (\(m\) and \(n\)) and \(0\) equations, C is most likely to be the answer. So, we should consider conditions 1) & 2) together first. After comparing the number of variables and the number of equations, we can save time by considering conditions 1) & 2) together first.

Conditions 1) & 2)
Since \(\frac{m}{n}\) is even, \(\frac{m}{n} = 2k\) and \(m = (2k)n\) for some positive integer \(k\).
If \(m = 2\) and \(n = 1\), then \(m + n = 3\), is an odd integer and the answer is ‘yes’.
If \(m = 4\) and \(n = 2\), then \(m + n = 6\) is an even integer and the answer is ‘no’.
Since we do not obtain a unique answer, conditions 1) & 2) are not sufficient when considered together.

Therefore, E is the answer.
Answer: E

In cases where 3 or more additional equations are required, such as for original conditions with “3 variables”, or “4 variables and 1 equation”, or “5 variables and 2 equations”, conditions 1) and 2) usually supply only one additional equation. Therefore, there is an 80% chance that E is the answer, a 15% chance that C is the answer, and a 5% chance that the answer is A, B or D. Since E (i.e. conditions 1) & 2) are NOT sufficient, when taken together) is most likely to be the answer, it is generally most efficient to begin by checking the sufficiency of conditions 1) and 2), when taken together. Obviously, there may be occasions on which the answer is A, B, C or D.

Originally posted by MathRevolution on 28 Dec 2018, 03:17.
Last edited by MathRevolution on 25 Jul 2021, 03:02, edited 1 time in total.
Math Revolution GMAT Instructor
Joined: 16 Aug 2015
Posts: 10161
Own Kudos [?]: 16600 [0]
Given Kudos: 4
GMAT 1: 760 Q51 V42
GPA: 3.82
Send PM
Re: Math Revolution DS Expert - Ask Me Anything about GMAT DS [#permalink]
Expert Reply
[Math Revolution GMAT math practice question]

(function) In the \(xy\)-coordinate plane, does \(y=a(x-h)^2+k\) intersect the \(x\)-axis?

\(1) h=1\)
\(2) k=2\)
Math Revolution GMAT Instructor
Joined: 16 Aug 2015
Posts: 10161
Own Kudos [?]: 16600 [0]
Given Kudos: 4
GMAT 1: 760 Q51 V42
GPA: 3.82
Send PM
Math Revolution DS Expert - Ask Me Anything about GMAT DS [#permalink]
Expert Reply
MathRevolution wrote:
[Math Revolution GMAT math practice question]

(number properties) If \(m\) and \(n\) are positive integers, is \(3^{4m+2}+n\) divisible by \(5\)?

\(1) m=3\)
\(2) n=1\)


=>

Forget conventional ways of solving math questions. For DS problems, the VA (Variable Approach) method is the quickest and easiest way to find the answer without actually solving the problem. Remember that equal numbers of variables and independent equations ensure a solution.

The first step of the VA (Variable Approach) method is to modify the original condition and the question. We then recheck the question.

The units digits of \(3^k\) have period \(4\) as they form the cycle \(3 -> 9 -> 7 -> 1.\)
\(3^{4m+2}\) has \(9\) as its units digit if \(3^{4m+2}\) has units digit \(9\), regardless of the value of \(m\).
Thus, the divisibility of \(3^{4m+2}+n\) by \(5\) relies on the variable n only.

Therefore, the correct answer is B.
Answer: B

Originally posted by MathRevolution on 30 Dec 2018, 18:28.
Last edited by MathRevolution on 25 Jul 2021, 03:03, edited 1 time in total.
Math Revolution GMAT Instructor
Joined: 16 Aug 2015
Posts: 10161
Own Kudos [?]: 16600 [0]
Given Kudos: 4
GMAT 1: 760 Q51 V42
GPA: 3.82
Send PM
Re: Math Revolution DS Expert - Ask Me Anything about GMAT DS [#permalink]
Expert Reply
MathRevolution wrote:
[Math Revolution GMAT math practice question]

(function) In the \(xy\)-coordinate plane, does \(y=a(x-h)^2+k\) intersect the \(x\)-axis?

\(1) h=1\)
\(2) k=2\)


=>

Forget conventional ways of solving math questions. For DS problems, the VA (Variable Approach) method is the quickest and easiest way to find the answer without actually solving the problem. Remember that equal numbers of variables and independent equations ensure a solution.

Since we have \(3\) variables (\(a, h\) and \(k\)) and \(0\) equations, E is most likely to be the answer. So, we should consider conditions 1) & 2) together first. After comparing the number of variables and the number of equations, we can save time by considering conditions 1) & 2) together first.

Conditions 1) & 2)
If \(a = 1\), then the graph doesn’t intersect the \(x\)-axis shown as below.

Attachment:
1231.png
1231.png [ 5.04 KiB | Viewed 1261 times ]


If \(a = -1\), then the graph intersects the \(x\)-axis shown as below.

Attachment:
12311.png
12311.png [ 5.23 KiB | Viewed 1282 times ]


Since neither condition gives us information about the value of \(a\), conditions 1) & 2) are not sufficient, when considered together.

Therefore, the answer is E.
Answer: E

In cases where 3 or more additional equations are required, such as for original conditions with “3 variables”, or “4 variables and 1 equation”, or “5 variables and 2 equations”, conditions 1) and 2) usually supply only one additional equation. Therefore, there is an 80% chance that E is the answer, a 15% chance that C is the answer, and a 5% chance that the answer is A, B or D. Since E (i.e. conditions 1) & 2) are NOT sufficient, when taken together) is most likely to be the answer, it is generally most efficient to begin by checking the sufficiency of conditions 1) and 2), when taken together. Obviously, there may be occasions on which the answer is A, B, C or D.
Math Revolution GMAT Instructor
Joined: 16 Aug 2015
Posts: 10161
Own Kudos [?]: 16600 [0]
Given Kudos: 4
GMAT 1: 760 Q51 V42
GPA: 3.82
Send PM
Re: Math Revolution DS Expert - Ask Me Anything about GMAT DS [#permalink]
Expert Reply
[Math Revolution GMAT math practice question]

(absolute value) Is \(\sqrt{(x+1)^2}=x+1\) ?

\(1) x(x-2) = 0\)
\(2) x(x+2) = 0\)
GMAT Club Bot
Re: Math Revolution DS Expert - Ask Me Anything about GMAT DS [#permalink]
   1  ...  4   5   6   7   8   9   10   11  ...  64   
Moderator:
Math Expert
92948 posts

Powered by phpBB © phpBB Group | Emoji artwork provided by EmojiOne